1. Standard memberSwissGambit
    Caninus Interruptus
    2014.05.01
    Joined
    11 Apr '07
    Moves
    92274
    18 Feb '08 19:53


    PG in 30.5 moves [find a legal game that reaches this position after White's 31st move]
  2. Joined
    12 Mar '03
    Moves
    44411
    18 Feb '08 22:041 edit
    It would be nice if we could have a few players combining efforts on this one. A few very first reactions :

    - three white knights, that means one of them is a promoted piece, from the original e2-pawn, that must have captured all three black missing pieces/pawn (three captures needed to go to the h-file; only three black pieces/pawns missing). Promotion on h8 (no 4th capture).

    - black's a-pawn captured white's white-coloured bishop with a6xBc5

    that's what I can see on first glance.

    white's last move was Qa5-a6+, not capturing anything
  3. Standard memberSwissGambit
    Caninus Interruptus
    2014.05.01
    Joined
    11 Apr '07
    Moves
    92274
    19 Feb '08 00:55
    Originally posted by Mephisto2
    It would be nice if we could have a few players combining efforts on this one.
    Calling peakite, david113 and ilywrin [though it's been a long time since he's posted in this forum!].
  4. Joined
    12 Mar '03
    Moves
    44411
    19 Feb '08 09:47
    I didn't spend much time but I think my first message already contained inaccuracies.

    If the e-pawn were the promoted one, then there were two imaginable paths: via e6xf7xg8=N or via e4xf5xg6xh7-h8=N. However both seem impossible. The first option would save on a capture & move(s) but there is no way black could get a rook on f7 to be captured. The second option fails because that would assume that the rook from h8 is captured on f5 or g6 BEFORE the h7-pawn is captured, but then the rook could not get out.

    Hence it was not the e-pawn that got promoted, but the f-pawn. It could go this way: f2-f4-f5xRg6xh7-h8=N. The black rook moves to g8 until the promoted knight left the promotion square, and then gets back out via h8 and towards f3 where it is captured by the e-pawn exRf3.

    Assuming the abovementioned points were correct, a next step is to count the minimum number of moves needed to put the white pieces at their final position & possible paths

    - King: 2 (e1-f2-g3)
    - Queen: 3 i.e. 2 to reach a5 (d1-h5-a5 or d1-a4-a5) and 1 last move (Qa5-a6)
    - Ra: 3 (Ra-e1 then up and one to the side, or Ra-a3 then 1 right and 1 up)
    - Rh: 3 (similar as above)
    - Bw: 1 (Bf1-b5)
    - Bb: 3 (Bc1-b2-e5-h2)
    - e-pawn: 1 (exRf3)
    - f-pawn 5 (f2-f4-f5 xg6 xh7 - h8=N)
    - promoted Knight 5 (for instance Nh8-f7-d6-c4-b6-a8 other paths possible too)
    - other pawns: 4

    Total: 30, leaving us only one spare move and lots of dependencies for the sequence

    other observations:
    - move Nb6-a8 must be made before b7-b6 by black, and after the rook got out to be captured on f5 or g6; that rook could only get out via a5 (after a6XBb5)

    - the black white-squared bishop could get out only after the promoted knight landed on a8, meaning also after a6xBb5, hence the bishop went via b7 to d6 to a2 (white has not the luxury to move the knight on b1 twice), and before the whire b- and c-pawns moved to their final position

    I hope to have a bit more time in the coming days, and to find input/comments by others (I may be wrong in some of my assumptions)
  5. Joined
    12 Mar '03
    Moves
    44411
    19 Feb '08 10:521 edit
    Still something wrong with the idea of promoting the f-pawn and having exRf3 later. I need a rook to be taken on g6 by the white pawn. But how to get the a-rook out before Bb5 and axb5 is played, when the e-pawn is still on its original square, blocking the white bishop?
  6. Joined
    30 Oct '04
    Moves
    7813
    19 Feb '08 14:012 edits
    True, I haven't posted here in a while. But I'll make an effort now - Black is missing 3 pieces - two Rooks and a pawn (three white captures). White is missing a White bishop (captured on "b5" most probably) and has the last pawn promoted to Knight. Only two ways to achieve that with three captures and they do not look too different (but I guess they are since one will obstruct the sequence of moves) the f pawn captures something on g file then captures another piece on the h file and promotes on h8, thus "e" pawn captures a black rook on f3. The other possible way - the e pawn makes all three captures and promotes to h8. The reasonable question is whether the knight at "a8" is the promoted one? No idea yet, though I suspect it's not. We will have to do some experiments with the quickest routes for the final positions of the three knights. Considerations - Black bishop on "a2" can only pass through b7, and the b-pawn has to move after the white knight (whichever it is) gets to "a8" via "b6". Which rook was captured on f3? The h or the a? Other than that , I believe it's White sequence of moves that will determine the play for the most part, probably with Black losing a tempo here and there (It takes Black about 20 moves to get to the final position). So what's the quickest setup for White? I'll think some more before adding anything.
    EDIT: Just read Mephisto's post - he makes some good points how does the "a" rook get freed if it must be captured on "f3' - looks impossible since the white bishop on "f1" is blocked. So it seems it must be the "e" pawn promoting after all. But how? It is immposible... Now alternatives - Black plays h5 -Rh6 -Rf6 -Rf3 - white here responds with exf and Bb5 freeing the black rook on "a" file which gets to a4 and g4, then White continues with fxg4 -gxh5, etc. But I am not sure if that does not make too many moves on white's part. Spare moves can only be h4, b3, Bb2, Be5 or else there will be obstruction to Black rooks paths. Will count and see.
    EDIT2: This will not work...Too many moves. Back to the drawing board.
  7. Joined
    30 Oct '04
    Moves
    7813
    19 Feb '08 15:252 edits
    Only possible way (working on the details):
    1) The bishop came to "a2" before the through c2 and b3 (sounds weird but see 2) and the knight at b1 has never moved
    2) The white queen came to a5 through a4 (after c4 was played) - the only way to get it in 31 moves
    3) b3 was played last (after c4, Qa4, Bc2, a4, Ra3, Rf3,Bb3,Ba2) so most probably last moves of white are Bb2-Be5-Bh2 (in any case after the previously mentioned sequence).
    4). White's first moves are most probably 1.h4 2.Rh3 3.Rd3 4.Rd6 Black's first moves are probably 1...h5 2...Rh6 3...Rf6 4...Rf3
    So we have a total of 31 white moves and some important sequence determined (11 pawn (including captures and promotion), 4 bishop, 6 rook, 3 queen, 5 knight (promoted only), 2 king)
    EDIT:
    1.h4 h5 2.Rh3 Rh6 3.Rd3 Rf6 4.Rd6 Rf3 5.exf3 a6 6.Bb5 axb5 7.c4 Ra3 8.Qa4 Rd3 9.Qa5 Rd4 10. a4 Rg4 11. fxg4 f6 12. gxh5 Kf7 13.h6 Kg6 14.h7 Kh6 15.h8N+ Kh7 16.Ng6 Nh6 17.Nf4 Kg8 18.Nd5 Kf7 19.Nb6 Ke8 20.Na8 b6 21.Ra3 Bb7 22.Rf3 Be4 23. Rf5 Bc2 24.f3 Bb3 25.Kf2 Ba2 26.b3 Nc6 27.Bb2 Qb8 28.Be5 Kd8 29.Bh2 Kc8 30. Kg3 Kb7 31.Qa6+
  8. Joined
    12 Mar '03
    Moves
    44411
    19 Feb '08 16:28
    Originally posted by ilywrin
    Only possible way (working on the details):
    1) The bishop came to "a2" before the through c2 and b3 (sounds weird but see 2) and the knight at b1 has never moved
    2) The white queen came to a5 through a4 (after c4 was played) - the only way to get it in 31 moves
    3) b3 was played last (after c4, Qa4, Bc2, a4, Ra3, Rf3,Bb3,Ba2) so most probably last moves of ...[text shortened]... Rf5 Bc2 24.f3 Bb3 25.Kf2 Ba2 26.b3 Nc6 27.Bb2 Qb8 28.Be5 Kd8 29.Bh2 Kc8 30. Kg3 Kb7 31.Qa6+
    Good work! Your second edit was needed to fix it (the little triangle Kg6-Kh6-Kh7 was necessary to make the timing of arriving at b7 work). The breakthrough for me is the black rook maneuvre via a3. I hadn't thought of that and was fixated on the sixth rank . Meanwhile I had figured out too that the pawn could be captured on h5 as well.
  9. Standard memberSwissGambit
    Caninus Interruptus
    2014.05.01
    Joined
    11 Apr '07
    Moves
    92274
    19 Feb '08 17:28
    Originally posted by ilywrin
    1.h4 h5 2.Rh3 Rh6 3.Rd3 Rf6 4.Rd6 Rf3 5.exf3 a6 6.Bb5 axb5 7.c4 Ra3 8.Qa4 Rd3 9.Qa5 Rd4 10. a4 Rg4 11. fxg4 f6 12. gxh5 Kf7 13.h6 Kg6 14.h7 Kh6 15.h8N+ Kh7 16.Ng6 Nh6 17.Nf4 Kg8 18.Nd5 Kf7 19.Nb6 Ke8 20.Na8 b6 21.Ra3 Bb7 22.Rf3 Be4 23. Rf5 Bc2 24.f3 Bb3 25.Kf2 Ba2 26.b3 Nc6 27.Bb2 Qb8 28.Be5 Kd8 29.Bh2 Kc8 30. Kg3 Kb7 31.Qa6+
    Yeah, that's it...composer is Michel Caillaud.
Back to Top

Cookies help us deliver our Services. By using our Services or clicking I agree, you agree to our use of cookies. Learn More.I Agree